LSAT and Law School Admissions Forum

Get expert LSAT preparation and law school admissions advice from PowerScore Test Preparation.

 Administrator
PowerScore Staff
  • PowerScore Staff
  • Posts: 8916
  • Joined: Feb 02, 2011
|
#31775
Please post below with any questions!
 jam1226
  • Posts: 6
  • Joined: May 22, 2017
|
#35723
Hi,

Could you please explain the problem with A? I see how E is correct, but prior to going through the answer choices I prephrased two possibilities:

1). slash reduce taxes on imports -> slash benefit many consumers (mistaken reversal of the first rule)
2). slash reduce taxes on imports -> Harm one or more domestic industry (repeat of the second rule)

I am just having trouble eliminating A.

Thank you!
 Jon Denning
PowerScore Staff
  • PowerScore Staff
  • Posts: 904
  • Joined: Apr 11, 2011
|
#35809
Hi jam,

Thanks for the question and welcome to the Forum!

For this Justify the Conclusion question we need to prove that "the government should not reduce taxes on textile imports" based on the following pair of principles:

..... 1. If reducing taxes on imports would financially benefit many domestic consumers the government should
..... reduce import taxes: Financially Benefit Consumers :arrow: Reduce Taxes

..... 2. An exception does exist: if a domestic industry would be significantly harmed by added competition the
..... government should NOT reduce import taxes: Harm Domestic Industry :arrow: NOT Reduce Taxes

So that second idea is how we're going to arrive at "not reduce taxes": we need to simply show that reducing import taxes would cause additional competition and harm some domestic industry.

That's what answer choice (E) gives us!

Answer choice (A) on the other hand plays on something we call a Mistaken Negation, where the absence of the sufficient (initial) condition is incorrectly taken to indicate something about the necessary condition. Specifically, when (A) tells us "NOT Financially Benefit Consumers," we've negated the sufficient part of the principle I numbered 1 above. But the absence of Financially Benefit Consumers gives us nothing! Without the trigger present—that is, without the condition that starts the relationship—we simply can't know anything. It's one of the classic conditional traps on the LSAT, and they're hoping you fall for it here...so remember, when the sufficient isn't there you have to stop. Go no further, you're done.

That's why (A) is incorrect in this question. We simply cannot know anything about what the government should or shouldn't do tax-wise until we have one of the two sufficient conditions I've listed above.

I hope that helps!
 jam1226
  • Posts: 6
  • Joined: May 22, 2017
|
#35828
Hi Jon,

Thank you for the detailed reply, and for clearing this issue up.

I think my mistake was thinking I could take the contrapositive of the first statement, and use the stated conclusion as a negated sufficient condition to arrive at answer choice A. Looking back on it now I overlooked this problem and moved to quickly thus selecting the trap answer A. It looks like this is a pretty straightforward "repeat" form conditional answer choice, and not too difficult of a question if I took a second.
User avatar
 relona
  • Posts: 24
  • Joined: Jul 23, 2021
|
#91870
Jon Denning wrote: Wed Jun 07, 2017 7:12 pm Hi jam,

Thanks for the question and welcome to the Forum!

For this Justify the Conclusion question we need to prove that "the government should not reduce taxes on textile imports" based on the following pair of principles:

..... 1. If reducing taxes on imports would financially benefit many domestic consumers the government should
..... reduce import taxes: Financially Benefit Consumers :arrow: Reduce Taxes

..... 2. An exception does exist: if a domestic industry would be significantly harmed by added competition the
..... government should NOT reduce import taxes: Harm Domestic Industry :arrow: NOT Reduce Taxes

So that second idea is how we're going to arrive at "not reduce taxes": we need to simply show that reducing import taxes would cause additional competition and harm some domestic industry.

That's what answer choice (E) gives us!

Answer choice (A) on the other hand plays on something we call a Mistaken Negation, where the absence of the sufficient (initial) condition is incorrectly taken to indicate something about the necessary condition. Specifically, when (A) tells us "NOT Financially Benefit Consumers," we've negated the sufficient part of the principle I numbered 1 above. But the absence of Financially Benefit Consumers gives us nothing! Without the trigger present—that is, without the condition that starts the relationship—we simply can't know anything. It's one of the classic conditional traps on the LSAT, and they're hoping you fall for it here...so remember, when the sufficient isn't there you have to stop. Go no further, you're done.

That's why (A) is incorrect in this question. We simply cannot know anything about what the government should or shouldn't do tax-wise until we have one of the two sufficient conditions I've listed above.

I hope that helps!
I don't completely understand why (A) could not be chosen. Isn't the necessary condition shown in the conclusion? Does it need to be shown in the answer choice as well? And if so, is it shown in (E)? I understand why (E) is correct though.
 Adam Tyson
PowerScore Staff
  • PowerScore Staff
  • Posts: 5153
  • Joined: Apr 14, 2011
|
#91882
Answer A is a Mistaken Negation of the first principle in the stimulus, relona. The rule there is:

Benefit Consumers :arrow: Reduce Taxes

We are looking for an answer that proves that we should NOT reduce taxes. There is no way to do that based on the first rule without making a Mistaken Negation. You can never prove that a Necessary Condition does not occur! You can only prove that it does occur (if the Sufficient Condition happens) or that the Sufficient Condition does not occur (if the Necessary Condition does not, which is the contrapositive).

To prove that we should NOT reduce taxes, we have to apply the second principle:

Harm :arrow: Reduce Taxes

To prove that Necessary Condition of NOT reducing taxes, we need an answer that shows significant harm to domestic industry. Only answer E does that!

Get the most out of your LSAT Prep Plus subscription.

Analyze and track your performance with our Testing and Analytics Package.